Source Latex: Exercices de mathématiques en Terminale S


Fichier
Type: Exercices
File type: Latex, tex (source)
Télécharger le document pdf compilé pdficon
Description
Exercices corrigés de mathématiques, Terminale S: probabilités continues
Niveau
Terminale S
Mots clé
probabilités continues, lois continues, loi uniforme, loi exponentielle, loi normale, exercices corrigés, maths, TS
Voir aussi:

Documentation sur LaTeX
lien vers la documentation Latex
Source LaTex icone

Source Latex

\documentclass[12pt,onecolumn,a4paper]{article}

\usepackage[french]{babel}
%\selectlanguage{francais}
\usepackage[utf8]{inputenc}
\usepackage{amsfonts}
\usepackage{amssymb}
\usepackage{amsmath}
\frenchbsetup{StandardLists=true}
\usepackage{enumerate}
\usepackage{array}
\usepackage{pst-all}
\usepackage{hyperref}
\hypersetup{
    pdfauthor={Yoann Morel},
    pdfsubject={Exercices corrigés de mathématiques: probabilités continues},
    pdftitle={Exercices corrigés de mathématiques: probabilités continues},
    pdfkeywords={probabilités, lois continues, exercices corrigés, Mathématiques, TS, terminale S}
}
\hypersetup{
    colorlinks = true,
    linkcolor = blue,
    anchorcolor = red,
    citecolor = blue,
    filecolor = red,
    urlcolor = red
}
% Raccourcis diverses:
\newcommand{\nwc}{\newcommand}
\nwc{\dsp}{\displaystyle}
\nwc{\bgar}{\begin{array}}\nwc{\enar}{\end{array}}
\nwc{\bgit}{\begin{itemize}}\nwc{\enit}{\end{itemize}}
\nwc{\bgen}{\begin{enumerate}}\nwc{\enen}{\end{enumerate}}

\nwc{\la}{\left\{}\nwc{\ra}{\right\}}
\nwc{\lp}{\left(}\nwc{\rp}{\right)}
\nwc{\lb}{\left[}\nwc{\rb}{\right]}

\nwc{\ul}{\underline}
\nwc{\tm}{\times}
\nwc{\V}{\overrightarrow}
\newcommand{\zb}{\mbox{$0\hspace{-0.67em}\mid$}}
\newcommand{\db}{\mbox{$\hspace{0.1em}|\hspace{-0.67em}\mid$}}
\newcommand{\ct}{\centerline}


\def\N{{\rm I\kern-.1567em N}}
\def\D{{\rm I\kern-.1567em D}}
\def\R{{\rm I\kern-.1567em R}}
\def\C{{\rm C\kern-4.7pt
\vrule height 7.7pt width 0.4pt depth -0.5pt \phantom {.}}}
\def\Q{\mathbb{Q}}
\def\Z{{\sf Z\kern-4.5pt Z}}
\def\euro{\mbox{\raisebox{.25ex}{{\it =}}\hspace{-.5em}{\sf C}}}

\newcounter{nex}[section]\setcounter{nex}{0}
\newenvironment{EX}{%
\stepcounter{nex}
\bigskip{\noindent{{\bf Exercice }}\arabic{nex}}\hspace{0.5cm}
}{}
\nwc{\bgex}{\begin{EX}}\nwc{\enex}{\end{EX}}

\nwc{\bgmp}{\begin{minipage}}\nwc{\enmp}{\end{minipage}}
\voffset=-3.2cm
\textheight=27.8cm
\textwidth=19.2cm
\topmargin=0cm
\headheight=-0.cm
\footskip=1.cm
\oddsidemargin=-1.5cm

\usepackage{fancyhdr}
\pagestyle{fancyplain}
\setlength{\headheight}{0cm}
\renewcommand{\headrulewidth}{0pt}
\renewcommand{\footrulewidth}{.1pt}
\lfoot{Y. Morel - \url{https://xymaths.fr/Lycee/TS/}}
\cfoot{}
\rfoot{Probabilités - Exercices corrigés - \thepage/\pageref{LastPage}}

\title{Probabilités - Exercices corrigés}
\author{Y. Morel}
%%%%%%%%%%%%%%%%%%%%%%%%%%%%%%%%%%%%%%%%%%%%%%%%%%%%%%%%%%%%%%%%%
\begin{document}

\maketitle



\noindent{\large\bf Exercice 1 }
Soit $X$ une variable aléatoire qui suit la loi uniforme sur 
$[-5;15]$. 
Calculer: 

\begin{enumerate}[a)] 
\item $P\lp X\leqslant 2\rp$ 

  \medskip\underline{Correction:}

  La fonction densité de probabilité de la loi uniforme sur $[-5;15]$
  est $f(x)=\dfrac{1}{15-(-5)}=\dfrac{1}{20}$, 

  et donc, 
  $\dsp 
  P\lp X\leqslant 2\rp
  =\int_{-5}^2 f(x)\,dx
  =\Bigl[\, \dfrac{x}{20}\,\Bigl]_{-5}^2
  =\dfrac{2}{20}-\dfrac{-5}{20}
  =\dfrac{7}{20}
  $ 


\item $P\lp -1\leqslant X\leqslant 1\rp$ 

  \medskip\underline{Correction:}

  De même qu'au a), 
  $\dsp 
  P\lp -1\leqslant X\leqslant 1\rp
  =\int_{-1}^1 f(x)\,dx
  =\Bigl[\, \dfrac{x}{20}\,\Bigl]_{-1}^1
  =\dfrac{1}{20}-\dfrac{-1}{20}
  =\dfrac{2}{20}
  =\dfrac{1}{10}
  $ 

\item $P_{\lp X\geqslant 0\rp}\lp -1\leqslant X\leqslant 2\rp$ 

  \medskip\underline{Correction:}

  $P_{\lp X\geqslant 0\rp}\lp -1\leqslant X\leqslant 2\rp
  =\dfrac{P\Bigl( \lp X\geqslant 0\rp\cap\lp-1\leqslant X\leqslant 2\rp\Bigr)}{P\lp X\geqslant 0\rp}
  =\dfrac{P\lp0\leqslant X\leqslant 2\rp}{P\lp X\geqslant 0\rp}
  =\dfrac{\dfrac{2}{20}}{\dfrac{15}{20}}
  =\dfrac{2}{15}
  $ 



\item Soit $Y$ la variable aléatoire égale à $\dfrac{X+5}{10}$. 
  Calculer $P_{\lp X\leqslant 10\rp}\lp Y\geqslant 1\rp$.

  \medskip\underline{Correction:}

  $\begin{array}{ll}
  P_{\lp X\leqslant 10\rp}\lp Y\geqslant 1\rp
  &=P_{\lp X\leqslant 10\rp}\lp \dfrac{X+5}{10}\geqslant 1\rp
  =P_{\lp X\leqslant 10\rp}\lp X+5\geqslant 10\rp
  =P_{\lp X\leqslant 10\rp}\lp X\geqslant 5\rp
  \\
  &=\dfrac{P\Bigl( \lp X\leqslant 10\rp\cap\lp X\geqslant 5\rp\Bigr)}{P\lp X\leqslant 10\rp}
  =\dfrac{P\lp 5\leqslant X\leqslant 10\rp}{P\lp X\leqslant 10\rp}
  =\dfrac{\dfrac{5}{20}}{\dfrac{15}{20}}
  =\dfrac{1}{3}
  \end{array}$
\end{enumerate}



\bigskip
\noindent{\large\bf Exercice 2 }
Soit $X$ une variable aléatoire qui suit la loi exponentielle de
paramètre $\lambda=3$. 
Calculer: 

\begin{enumerate}[a)] 
\item $P\lp X\leqslant 2\rp$ 

  \medskip\underline{Correction:} 

  La fonction densité de probabilité de la loi exponentielle de
  paramètre $\lambda=3$ est 
  \mbox{$f(x)=\lambda e^{-\lambda x}=3\,e^{-3x}$}, 

  et donc, 
  $\dsp 
  P\lp X\leqslant 2\rp
  =\int_0^2 3e^{-3x}\,dx
  =\Bigl[\, -e^{-3x}\,\Bigr]_0^2
  =-e^{-3\tm2}+e^{-3\tm0}=-e^{-6}+1
  $ 

\item $P\lp X\geqslant 4\rp$ 

  \medskip\underline{Correction:} 

  $\dsp 
  P\lp X\geqslant 4\rp
  =1-P\lp X<4\rp
  =1-\int_0^4 3e^{-3x}\,dx
  =1-\Bigl[\, -e^{-3x}\,\Bigr]_0^4
  =1-\Bigl( -e^{-12}+e^{0}\Bigr)
  =e^{-12}
  $ 

\item $P\lp 2\leqslant X\leqslant 4\rp$ 

  \medskip\underline{Correction:} 

  $\dsp 
  P\lp 2\leqslant X\leqslant 4\rp
  =\int_2^4 3e^{-3x}\,dx
  =\Bigl[\, -e^{-3x}\,\Bigr]_2^4
  =-e^{-12}+e^{-6}
  $ 

\item $P_{\lp X\geqslant 2\rp}\lp X\geqslant 4\rp$ 

  \medskip\underline{Correction:} 
  
  $P_{\lp X\geqslant 2\rp}\lp X\geqslant 4\rp
  =\dfrac{P\Bigl( \lp X\geqslant 2\rp\cap\lp X\geqslant 4\rp\Bigr)}{P\lp X\geqslant 2\rp}
  =\dfrac{P\lp X\geqslant 4\rp}{P\lp X\geqslant 2\rp}
  =\dfrac{e^{-12}}{e^{-6}}
  =e^{-12+6}=e^{-6}
  $ 

\item $P_{\lp X\geqslant 122\rp}\lp X\geqslant 124\rp$ 

  \medskip\underline{Correction:} 
  
  $P_{\lp X\geqslant 122\rp}\lp X\geqslant 124\rp
  =\dfrac{P\Bigl( \lp X\geqslant 122\rp\cap\lp X\geqslant 124\rp\Bigr)}{P\lp X\geqslant 122\rp}
  =\dfrac{P\lp X\geqslant 124\rp}{P\lp X\geqslant 122\rp}
  =\dfrac{e^{-3\tm 124}}{e^{-3\tm 122}}
  =e^{-6}
  $ 

\item Soit deux réels $a>0$ et $h>0$. 
  Montrer que la probabilité 
  $P_{\lp X\geqslant a\rp}\lp X\geqslant a+h\rp$ ne dépend pas de~$a$. 

  \medskip\underline{Correction:} 
  
  $P_{\lp X\geqslant a\rp}\lp X\geqslant a+h\rp
  =\dfrac{P\Bigl( \lp X\geqslant a\rp\cap\lp X\geqslant a+h\rp\Bigr)}{P\lp X\geqslant a\rp}
  =\dfrac{P\lp X\geqslant a+h\rp}{P\lp X\geqslant a\rp}
  $

  avec, 
  $\dsp 
  P\lp X\geqslant a\rp
  =1-P\lp X<a\rp
  =1-\int_0^{a} 3e^{-3x}\,dx
  =1-\Bigl[\,-e^{-3x}\,\Bigr]_0^a
  =1-\Bigl( -e^{-3a}+1\Bigr)
  =e^{-3a}
  $

  et de même, 
  $P\lp X\geqslant a\rp=e^{-3(a+h)}$, 

  d'où, 
  $P_{\lp X\geqslant a\rp}\lp X\geqslant a+h\rp
  =\dfrac{P\lp X\geqslant a+h\rp}{P\lp X\geqslant a\rp}
  =\dfrac{e^{-3\tm(a+h)}}{e^{-3\tm a}}
  =e^{-3\tm(a+h)+3\tm a}
  =e^{-3a}
  $. 

  Cette probabilité ne dépend donc effectivement pas de $a$.  
\end{enumerate}

\bigskip
\noindent{\large\bf Exercice 3 }
Soit $X$ une variable aléatoire qui suit la loi normale 
$\mathcal{N}\lp 500;20^2\rp$. 

\noindent
Pour $Z$ une variable aléatoire qui suit la loi normale centrée
réduite %$\mathcal{N}(0;1)$
, on note et donne 
$a=P\lp Z\leqslant 0\rp$, 
$b=P\lp Z\leqslant 0,5\rp\simeq, 0,6915$, 
$c=P\lp Z\leqslant 1\rp\simeq 0,8413$, 
$d=P\lp Z\leqslant 2\rp\simeq 0,9772$, 

\medskip
Exprimer en fonction de $a$, $b$, $c$ et $d$, puis donner une valeur
approchée de: 

\begin{enumerate}[a)] 
\item $P\lp X\leqslant 520\rp$

  \medskip\underline{Correction:} 

  Le calcul peut se faire directement à la calculatrice 
  (à utiliser donc pour vérifier le résultat), mais ici on doit 
  exprimer cette probabilité en fonction des données $a$, $b$, $c$ et
  $d$ de l'énoncé. 

  On doit donc se ramener à la loi normale centrée réduite. 

  Soit la variable aléatoire $Z=\dfrac{X-500}{20}$; 
  alors $Z$ suit la loi normale centrée réduite $\mathcal{N}(0;1)$, 
  et 

  $P\lp X\leqslant 520\rp
  =P\lp \dfrac{X-500}{20} \leqslant \dfrac{520-500}{20}\rp
  =P\lp Z\leqslant 1\rp 
  =c\simeq 0,8413
  $

\item $P\lp X\geqslant 540\rp$

  \medskip\underline{Correction:} 

  $P\lp X\geqslant 540\rp
  =1-P\lp X<540\rp
  =1-P\lp \dfrac{X-500}{20}<\dfrac{540-500}{20}\rp
  =1-P\lp Z<2\rp
  =1-d\simeq 0,0228
  $

  (car $P\lp Z<2\rp=P\lp Z\leqslant 2\rp$, pour $Z$ une variable
  aléatoire {\bf continue}). 

\item $P\lp 460\leqslant X\leqslant 540\rp$

  \medskip\underline{Correction:} 

  $\begin{array}{ll}
  P\lp 460\leqslant X\leqslant 540\rp
  &=P\lp \dfrac{460-500}{20}\leqslant \dfrac{X-500}{20}
  \leqslant \dfrac{540-500}{20}\rp
  =P\lp -2\leqslant Z\leqslant 2\rp
  \\
  &=P\lp Z\leqslant 2\rp - P\lp Z\leqslant -2\rp
  =P\lp Z\leqslant 2\rp - \Bigl( 1-P\lp Z\leqslant 2\rp\Bigr)
  \\
  &=d-\Bigl(1-d\Bigr)
  =2d-1\simeq 0,9544
  \end{array}$

\item $P_{\lp X\geqslant 500\rp}\lp X\leqslant 510\rp$

  \medskip\underline{Correction:} 
  
  $P_{\lp X\geqslant 500\rp}\lp X\leqslant 510\rp
  =\dfrac{P\Bigl(\lp X\geqslant 500\rp\cap\lp X\leqslant 510\rp\Bigr)}{P\lp X\geqslant 500\rp} 
  =\dfrac{P\lp 500\leqslant X\leqslant 510\rp}{P\lp X\geqslant 500\rp} 
  $

  avec, 
  \[\begin{array}{ll}
  P\lp 500\leqslant X\leqslant 510\rp
  &=P\lp \dfrac{500-500}{20}\leqslant \dfrac{X-500}{20}\leqslant
  \dfrac{510-500}{20}\rp
  \\[0.3cm]
  &=P\lp 0\leqslant Z\leqslant 0,5\rp
  \\[0.3cm]
  &=P\lp Z\leqslant 0,5\rp-P\lp Z\leqslant 0\rp
  =b-a
  \end{array}\]

  et 
  $
  P\lp X\geqslant 500\rp
  =P\lp \dfrac{X-500}{20}\geqslant \dfrac{500-500}{20}\rp
  =P\lp Z\geqslant 0\rp
  =1-P\lp Z<0\rp
  =1-a
  $. 

  Ainsi, 
  $P_{\lp X\geqslant 500\rp}\lp X\leqslant 510\rp
  =\dfrac{b-a}{1-a}\simeq 0,383
  $

  (On se rappelle pour ce dernier calcul que, la loi normale centrée
  réduite est symétrique, et donc, 
  $a=P\lp Z\leqslant 0\rp=P\lp Z\geqslant 0\rp=0,5$). 
\end{enumerate}



\bigskip
\noindent{\large\bf Exercice 4 }
Soit $X$ une variable aléatoire suivant la loi normale 
$\mathcal{N}\lp 200; 15^2\rp$. 

Déterminer le réel $u>0$ tel que 
$P\lp 200-2u\leqslant X \leqslant 200+2u\rp = 0,9$.

\medskip\noindent

\medskip\underline{Correction:} 

On se ramène à la loi normale centrée réduite: 
soit la variable aléatoire $Y=\dfrac{X-200}{15}$ qui suit donc la loi 
$\mathcal{N}(0;1)$, alors 

\[\begin{array}{ll}
P\lp 200-2u\leqslant X \leqslant 200+2u\rp = 0,9
&\iff 
P\lp \dfrac{200-2u-200}{15}\leqslant \dfrac{X-200}{15}\leqslant 
\dfrac{200+2u-200}{15}\rp=0,9
\\
&\iff 
P\lp -\dfrac{2u}{15}\leqslant Y\leqslant \dfrac{2u}{15}\rp=0,9
\\
&\iff 
P\lp Y\leqslant \dfrac{2u}{15}\rp-P\lp Y\leqslant -\dfrac{2u}{15}\rp=0,9
\\
&\iff 
P\lp Y\leqslant \dfrac{2u}{15}\rp-\Bigl(1-P\lp Y\leqslant \dfrac{2u}{15}\rp\Bigr)=0,9
\\
&\iff 
2\tm P\lp Y\leqslant \dfrac{2u}{15}\rp-1=0,9
\\
&\iff P\lp Y\leqslant \dfrac{2u}{15}\rp=\dfrac{1+0,9}{2}=0,95
\end{array}\]

A l'aide de la calculatrice, ou de la table de valeurs de la loi
normale centré réduite, on trouve que 
$P\lp Y\leqslant 1,65\rp\simeq 0,95$. 

On doit donc avoir 
$\dfrac{2u}{15}\simeq 1,65
\iff u\simeq \dfrac{1,65\tm 15}{2}\simeq 12,375
$



\bigskip
\noindent{\large\bf Exercice 5 }
Soit $X$ une variable aléatoire suivant la loi normale 
$\mathcal{N}\lp \mu; \sigma^2\rp$. 

On donne $\mu=E(X)=120$. 

Déterminer l'écart-type $\sigma$ tel que 
$P\lp 100\leqslant X \leqslant 140\rp=0,92$. 


\medskip\underline{Correction:} 

On se ramène à la loi normale centrée réduite: 
soit la variable aléatoire $Y=\dfrac{X-120}{\sigma}$ qui suit donc la loi 
$\mathcal{N}(0;1)$, alors 

$\begin{array}{ll}
P\lp 100\leqslant X \leqslant 140\rp=0,92
&\iff
P\lp \dfrac{100-120}{\sigma}\leqslant\dfrac{X-120}{\sigma}
\leqslant \dfrac{140-120}{\sigma}\rp=0,92
\\
&\iff
P\lp -\dfrac{20}{\sigma}\leqslant Y\leqslant \dfrac{20}{\sigma}\rp=0,92
\\
&\iff
P\lp Y\leqslant \dfrac{20}{\sigma}\rp-P\lp Y\leqslant -\dfrac{20}{\sigma}\rp-=0,92
\\
&\iff
P\lp Y\leqslant \dfrac{20}{\sigma}\rp-\Bigl(1-P\lp Y\leqslant \dfrac{20}{\sigma}\rp\Bigr)=0,92
\\
&\iff
2 P\lp Y\leqslant \dfrac{20}{\sigma}\rp-1=0,92
\\
&\iff
P\lp Y\leqslant \dfrac{20}{\sigma}\rp=\dfrac{0,92+1}{2}=0,96
\end{array}$. 

A l'aide de la calculatrice, ou de la table de valeurs de la loi
normale centré réduite, on trouve que 
$P\lp Y\leqslant 1,76\rp\simeq 0,96$, 

et on doit donc avoir 
$\dfrac{20}{\sigma}\simeq 1,76
\iff
\sigma\simeq \dfrac{20}{1,76}\simeq 11,36
$.



\bigskip
\noindent{\large\bf Exercice 6 }
{\it Surréservation d'une compagnie aérienne}

Une compagnie utilise des avions d'une capacité de 320 passagers. 
Une étude statistique montre que 5 passagers sur 100 ayant réservé ne
se présente pas à l'embarquement. 
On considérera ainsi que la probabilité qu'un passager ayant réservé
ne se présente pas à l'embarquement est de 0,05. 

\begin{enumerate}
\item La compagnie accepte 327 réservations sur un vol. 
  
  Soit $X$ la variable aléatoire indiquant le nombre de passagers se
  présentant à l'embarquement. 

  \begin{enumerate}[a.] 
  \item Quelle est la loi de probabilité suivie par $X$ ? 

    \medskip\underline{Correction:} 

    On répète $n=327$ fois le tirage aléatoire d'un passager. 
    C'est une épreuve de Bernoulli dont le succès est 
    "le passager se présente à l'embarquement", événement dont la
    probabilité est $p=1-0,05=0,95$. 

    Ces répétitions sont identiques et indépendantes 
    (on suppose que chaque personne se présente ou non à
    l'embarquement indépendamment du choix des autres passagers). 

    La variable aléatoire $X$ qui compte le nombre de passagers se
    présentant à l'embarquement, c'est-à-dire le nombre de succès dans
    les 327 répétitions, suit donc la loi binomiale
    $\mathcal{B}(327;0,95)$. 


  \item Par quelle loi normale peut-on approcher la loi de $X$ ? 
    Les paramètres de la loi seront déterminés à $10^{-2}$ près. 

    \medskip\underline{Correction:} 

    Comme $n=327\geqslant 30$, $np=310,95\geqslant 5$ et
    $n(1-p)=16,35\geqslant 5$, 
    d'après le théorème de Moivre-Laplace, la loi de probabilité de
    $X$ peut-être approchée par la loi normale de paramètre 
    $\mu=np= 310,65$ et d'écart-type 
    $\sigma=\sqrt{np(1-p)}\simeq 3,94$. 

    
  \item En utilisant l'approximation par la loi normale, 
    calculer $P(X\leqslant 320)$. 

    Penser vous que le risque pris par la compagnie en acceptant 327
    réservations soit important ? 

    \medskip\underline{Correction:} 
    
    Avec cette approximation, 
    \[\begin{array}{ll}
    P(X\leqslant 320)
    &\simeq P\lp \dfrac{X-310,65}{3,94}\leqslant \dfrac{320-310,65}{3,94}\rp
    \simeq P\lp \dfrac{X-310,65}{3,94}\leqslant 2,37\rp
    \\[0.4cm]
    &\simeq \Pi(2,37)
    \simeq 0,99
    \end{array}\]

    où on a utilisé la fonction de répartition de la loi normale
    centrée réduite $\Pi$ (dont les valeurs sont données dans la table
    ou calculées par la calculatrice). 

    \medskip
    Le risque pris par la compagnie d'avoir plus de passagers qui
    présentent à l'embarquement que de places réellement disponible
    est faible, il est inférieur à 1\,\%. 
  \end{enumerate}

\item Serait-il raisonnable pour la compagnie d'accepter sur ce même
  vol 330 réservations ? 335 réservations ?

  \medskip\underline{Correction:} 

  En procédant de même, 
  on trouve avec 330 réservations: 
  \[P\lp X\leqslant 320\rp
  \simeq P\lp \dfrac{X-313,5}{3,96}\leqslant \dfrac{320-313,5}{3,96}\rp
  \simeq P\lp \dfrac{X-313,5}{3,96}\leqslant 1,64\rp
  \simeq \Pi(1,64)
  \simeq 0,95 
  \]


  et, avec 335 réservations: 

  \[P\lp X\leqslant 320\rp
  \simeq P\lp \dfrac{X-318,25}{3,99}\leqslant \dfrac{320-318,25}{3,99}\rp
  \simeq P\lp \dfrac{X-318,25}{3,99}\leqslant 0,44\rp
  \simeq \Pi(0,44)
  \simeq 0,67
  \] 

  Ainsi, avec 330 réservations, le risque qu'il y ait plus de
  passagers se présentant à l'embarquement que de places disponibles
  reste inférieur à 5\,\%, tandis qu'avec 335 réservations ce risque
  devient de l'ordre de 33\,\% (environ 1 chance sur 3). 

  Ce dernier cas paraît alors déjà bien moins raisonnable. 


\item La compagnie accepte 337 réservation sur ce même vol d'une
  capacité de 320 passagers. 

  310 personnes sont déjà présentes à l'embarquement. Quelle est la
  probabilité que moins de 320 personnes se présentent en tout à
  l'embarquement ?

  
  \medskip\underline{Correction:} 

  En procédant de même que précédemment, avec 337 réservations, on
  recherche la probabilité conditionnelle: 

  \[
  P_{\lp X\geqslant 310\rp}\lp X\leqslant 320\rp
  =\dfrac{P\Bigl( \lp X\geqslant 310\rp\cap\lp X\leqslant 320\rp\Bigr)}{P\lp X\geqslant 310\rp}
  =\dfrac{P\lp 310\leqslant X\leqslant 320\rp}{P\lp X\geqslant 310\rp}
  \]
  avec,  
  \[\begin{array}{ll}
  P\lp 310\leqslant X\leqslant 320\rp
  &\simeq P\lp \dfrac{310-320,15}{4}\leqslant \dfrac{X-320,15}{4}
  \leqslant \dfrac{320-320,15}{4}\rp
  \\[0.3cm]
  &\simeq P\lp -2,54\leqslant \dfrac{X-320,15}{4}\leqslant -0,04\rp
  \\[0.3cm]
  &\simeq \Pi(-0,04)-\Pi(-2,54)
  \\[0.2cm]
  &\simeq 1-\Pi(0,04)-\lp 1 - \Pi(2,54)\rp
  \\[0.2cm]
  &\simeq \Pi(2,54)-\Pi(0,04)
  \simeq 0,478
  \end{array}\]

  et de même, 
  \[\begin{array}{ll}
  P\lp X\geqslant 310\rp
  &\simeq P\lp \dfrac{X-320,15}{4} \geqslant \dfrac{310-320,15}{4}\rp
  \simeq P\lp \dfrac{X-320,15}{4} \geqslant -2,54\rp
  \\[0.3cm]
  &\simeq 1-\Pi(-2,54)
  \simeq 1-\lp 1-\Pi(2,54)\rp
  \\[0.3cm]
  &\simeq \Pi(2,54)
  \simeq 0,994
  \end{array}\] 

  Au final, on a donc, 
  $P_{\lp X\geqslant 310\rp}\lp X\leqslant 320\rp\simeq 
  \dfrac{0,478}{0,994}\simeq 0,48$. 
\end{enumerate}


\bigskip
\noindent{\large\bf Exercice 7 }
Une entreprise fabrique des brioches en grande quantité. 

On pèse les boules de pâte avant cuisson. 
On note $X$ la variable aléatoire qui, à chaque boule de pâte, associe
sa masse. 
On admet que $X$ suit la loi normale de moyenne 700 g et d'écart type
20 g. 

\begin{enumerate}
\item Seules les boules dont la masse est comprise entre 666 g et 732
  g sont acceptées à la cuisson. 

  Quelle est la probabilité qu'une boule, prise au hasard dans la
  production, soit acceptée à la cuisson ? 


  \medskip\underline{Correction:} 
  
  Une boule est acceptée à la cuisson si 
  $\lp 666\leqslant X\leqslant 732\rp$ dont la probabilité est: 
  \[
  \lp 666\leqslant X\leqslant 732\rp
  =P\Bigl( \dfrac{666-700}{20}\leqslant \dfrac{X-700}{20}
  \leqslant \dfrac{32-700}{20}\Bigr)
  = P\Bigl( -1,7 \leqslant Y \leqslant 1,6\Bigr)
  \]
  où $Y=\dfrac{X-700}{20}$ est une variable aléatoire suivant la loi
  normale centrée réduite $\mathcal{N}(0;1)$, 
  et donc, en notant $\Pi$ sa fonction dé répartion, 
  \[
  \lp 666\leqslant X\leqslant 732\rp
  =\Pi(1,6)-\Pi(-1,7)
  =\Pi(1,6)-\lp 1- \Pi(1,7)\rp
  =\Pi(1,6)+\Pi(1,7)-1
  \simeq 0,90
  \]

  {\it Remarque: le calcul de $\lp 666\leqslant X\leqslant 732\rp$
    peut aussi se faire directement à l'aide de la calculatrice,
    néanmoins, cette démarche est à connaître et est de plus
    incontournable pour la question suivante.) }


\item Déterminer le réel positif $h$ afin que l'on
  ait: 
  $P(700-h\leqslant X\leqslant 700+h)\geqslant 0,95$. 

  Enoncer ce résultat à l'aide d'une phrase. 

  \medskip\underline{Correction:} 

  Avec les mêmes notations qu'à la question précédente: 
  \[\begin{array}{ll}
  P(700-h\leqslant X\leqslant 700+h)
  &= P\lp \dfrac{700-h-700}{20}\leqslant \dfrac{X-700}{20}
  \leqslant \dfrac{700+h-700}{20}\rp 
  \\
  &=P\lp -\dfrac{h}{20}\leqslant Y \leqslant \dfrac{h}{20}\rp
  \\
  &=\Pi\lp\dfrac{h}{20}\rp-\Pi\lp-\dfrac{h}{20}\rp
  \\
  &=\Pi\lp\dfrac{h}{20}\rp-\lp1-\Pi\lp\dfrac{h}{20}\rp\rp
  \\
  &=2\Pi\lp\dfrac{h}{20}\rp-1
  \end{array}\]

  et ainsi, 
  \[\begin{array}{ll}
  P(700-h\leqslant X\leqslant 700+h)\geqslant 0,95
  &\iff 
  2\Pi\lp\dfrac{h}{20}\rp-1\geqslant 0,95 
  \\[0.3cm]
  &\iff 
  \Pi\lp\dfrac{h}{20}\rp\geqslant \dfrac{0,95+1}{2}=0,975
  \end{array}\]

  A l'aide de la table des valeurs de $\Pi$ ou de la calculatrice, on
  trouve que $\Pi(x)\geqslant 0,975$ dès que $x\geqslant 1,89$. 

  On doit donc avoir 
  $\dfrac{h}{20}\simeq 1,89
  \iff
  h\simeq 1,89\tm 20\simeq 37,8$. 

  \medskip
  Ce résultat siginifie que plus de 95\,\% des boules de pâte ont 
  une masse comprise entre $700-h\simeq 662,2$g et 
  $700+h\simeq 737,8$g. 


\item On admet que 8\% des boules sont refusées à la cuisson. 
  On prélève au hasard, successivement et avec remise, $n$ boules dans
  la production. 
  On note $Y_n$ la variable aléatoire qui, à chaque prélèvement de $n$
  boules, associe le nombre de boules qui seront refusées à la
  cuisson. 
  Cette variable aléatoire $Y_n$ suit une loi binomiale. 

  Dans le cas $n=10$, 
  \begin{enumerate}[a.]
  \item calculer la probabilité d'avoir, parmi les
    10 boules prélevées, exactement 3 boules refusées à la cuisson; 


    \medskip\underline{Correction:} 

    $Y_{10}$ suit la loi binomiale 
    $\mathcal{B}(10;0,08)$, et donc 
    la probabilité d'avoir, parmi les
    10 boules prélevées, exactement 3 boules refusées à la cuisson,
    est: 
    \[
    P\lp Y_{10}=3\rp
    =\binom{10}{3} p^3 (1-p)^7
    \simeq 0,034
    \]


  \item calculer la probabilité d'avoir, parmi les 10 boules
    prélevées, au moins 7 boules acceptées à la cuisson. 

    \medskip\underline{Correction:} 

    La probabilité d'avoir, parmi les 10 boules
    prélevées, au moins 7 boules acceptées à la cuisson, 
    soit aussi strictement moins de 3 boules refusées, est: 
    \[\begin{array}{ll}
    P\lp Y_{10}<3\rp
    &=P\lp Y_{10}=0\rp+P\lp Y_{10}=1\rp+P\lp Y_{10}=2\rp
    \\[0.3cm]
    &=\binom{10}{0}p^0(1-p)^{10}+\binom{10}{1}p^1(1-p)^9+\binom{10}{2}p^2(1-p)^8
    \\[0.4cm]
    &\simeq 0,96
    \end{array}\]

  \end{enumerate}
\end{enumerate}



\bigskip
\noindent{\large\bf Exercice 8 } 
Une ligne de transmission entre un émetteur et un récepteur transporte
des pages de texte, chaque page étant représentée par 100\,000 bits. 

La probabilité pour qu'un bit soit erroné est estimé à 0,0001 et on
admet que les erreurs sont indépendantes les unes des autres. 

\medskip\noindent
{\bf Partie A.} Soit $X$ la variable aléatoire donnant le nombre
d'erreurs lors de la transmission d'une page. 

\begin{enumerate}
\item Quelle est la loi de probabilité suivie par $X$ ? 
  
  Calculer la moyenne et l'écart type de $X$. 

  \medskip\underline{Correction:} 

  Pour transmettre une page, on répète $n=100\,000$ fois la
  transmission d'un bit, de manière identique et indépendante. 

  La variable aléatoire $X$ compte le nombre de bits erronés, ce qui
  arrive avec la probabilité $p=10^{-4}$, 
  sur ces $100\,000$ répétitions. 
  $X$ suit donc la loi binomiale $\mathcal{B}(10^5;10^{-4})$. 

  Sa moyenne est donc $E(X)\!=\!np=10^5\tm 10^{-4}\!=\!10$, 
  et son écart-type \mbox{$\sigma(X)=\sqrt{np(1-p)}\simeq 3,16$}. 



\item On admet que cette loi peut être approchée par une loi normale
  de paramètres $m=10$ et $\sigma=\sqrt{10}$. 
  Dans ces conditions, déterminer la probabilité pour qu'une page
  comporte au plus 15 erreurs. 


  \medskip\underline{Correction:} 

  La probabilité qu'une page comporte au plus 15 erreurs et alors 
  \[P\lp X\leqslant 15\rp
  =P\lp \dfrac{X-10}{\sqrt{10}}\leqslant \dfrac{15-10}{\sqrt{10}}\rp
  \simeq P\lp U\leqslant 1,58\rp
  \]
  où la variable aléatoire $U=\dfrac{X-10}{\sqrt{10}}$ suit la loi
  normale centrée réduite $\mathcal{N}(0;1)$, et donc, 
  \[P\lp X\leqslant 15\rp
  \simeq 0,94
  \]
\end{enumerate}

\medskip\noindent
{\bf Partie B.} Pour corriger les erreurs commises à la suite de la
transmission d'une page, on transmet cette page autant de fois qu'il
le faut jusqu'à l'obtention d'une page sans erreur. 

Soit $Y$ la variable aléatoire égale au nombre de transmissions (d'une
même page) nécessaires pour obtenir une page sans erreur. 
On suppose que $p=0,05$ est la probabilité de transmission d'une page
sans erreur et 
$q=1-p$ est la probabilité de transmission d'une page avec erreur. 

On admet que $Y$ suit la loi de probabilité $P$ définie par 
$P(Y=n)=pq^{n-1}$; pour tout $n$ entier naturel non nul. 

\begin{enumerate}[a.]
\item Calculer $P(Y\leqslant 5)$.

  \medskip\underline{Correction:} 

  On cherche la probabilité: 
  \[\begin{array}{ll}
  P(Y\leqslant 5)
  &=P(Y=1)+P(Y=2)+P(Y=3)+P(Y=4)+P(Y=5)
  \\[0.3cm]
  &=pq^0+pq^1+pq^2+pq^3+pq^4
  \\[0.3cm]
  &\simeq 0,23
  \end{array}\]

\item Montrer que pour tout entier $n\geqslant1$,  
  $P(Y\leqslant n)=1-q^n$. 

  \medskip\underline{Correction:} 
  
  \[\begin{array}{ll}
  P(Y\leqslant n)
  &=P(Y=1)+P(Y=2)+P(Y=3)+\cdots+P(Y=n)
  \\[0.3cm]
  &=pq^0+pq^1+pq^2+\cdots+pq^{n-1}
  \\[0.3cm]
  &=p\lp q^0+q^1+q^2+\cdots+q^{n-1}\rp
  \end{array}\]
  or, $q^0+q^1+q^2+\cdots+q^{n-1}$ est la somme des premiers termes
  d'une suite géométrique de raison~$q$, soit 
  $\dfrac{1-q^n}{1-q}=\dfrac{1-q^n}{p}$, car $p=1-q$, 
  et donc, finalement on a bien, 
  \[P(Y\leqslant n)= p \dfrac{1-q^n}{p} = 1-q^n\]
\end{enumerate}



\bigskip
\noindent{\large\bf Exercice 9 }
On souhaite connaître le nombre de poissons vivants dans un lac clos. 
Pour cela, on prélève 500 poissons au hasard dans ce lac, on les
marque puis on les relâche dans le lac. 

Quelques jours plus tard, on prélève à nouveau aléatoirement 500
poissons dans le lac. 
Parmi ces 500 poissons, on en compte 24 qui sont marqués. 

On suppose que pendant la période d'étude le nombre $N$ de poissons dans
le lac est stable. 

\begin{enumerate}
\item Quelles sont les proportions $p$ et $p'$ de poissons marqués
  dans l'échantillon prélevé et dans le lac ?

  \medskip\underline{Correction:} 

  On a compté 24 poissons marqués sur l'échantillon de 500, 
  $p=\dfrac{24}{500}=0,048=4,8\,\%$, tandis que dans le lac il y a en
  tout 500 poissons marqués soit $p'=\dfrac{500}{N}$. 


\item Donner, à $10^{-3}$ près, l'intervalle de confiance au niveau de
  95\,\% de la proportion de poissons marqués dans le lac. 

  \medskip\underline{Correction:} 

  L'intervalle de confiance au niveau de 95\,\% pour une proportion
  $p=0,048$ dans un échantillon de taille $n=500$ est: 
  \[
  I=\Bigl[\ p-\dfrac{1}{\sqrt{n}}\ ;\ p+\dfrac{1}{\sqrt{n}}\ \Bigr]
  =\Bigl[\ 0,048-\dfrac{1}{\sqrt{500}}\ ;\ 0,048+\dfrac{1}{\sqrt{500}}\ \Bigr]
  \simeq \Bigl[\  0,003\ ;\ 0,093\ \Bigr]
  \]


\item En déduire un encadrement de la proportion du nombre de poissons 
  dans le lac puis du nombre de poissons dans le lac. 

  \medskip\underline{Correction:} 

  L'intervalle de confiance précédent est un encadrement pour la
  proportion $p'$ dans la population complète (ici tout le lac), 
  et donc, on a 
  \[
  0,003\leqslant p'=\dfrac{500}{N}\leqslant 0,093 
  \iff 
  \dfrac{500}{0,093}\leqslant N\leqslant \dfrac{500}{0,003}
  \iff 
  5\,376\leqslant N\leqslant 166\,666
  \]

  La proportion $p'$ de poissons marqués dans le lac est comprise
  entre $0,3\,\%$ et $9,3\,\%$, et le nombnre de poissons est compris
  entre $5\,376$ et $166\,666$ (à un niveau de confiance de 95\,\%). 


\item On considère que la population de poissons est trop importante
  pour le lac (dimensions, ressources, \dots ) lorsqu'il y a plus de
  50\,000 poissons qui y vivent. 

  En supposant que la proportion $p$ de poissons marqués 
  reste la même dans un échantillon prélevé de plus grande taille,
  quelle devrait-être cette taille pour que l'on puissse affirmer, au
  niveau de confiance de 95\,\%, que le lac n'est pas surpeuplé en
  poissons ?

  \medskip\underline{Correction:} 

  On cherche la taille $n$ de l'échantillon de manière à pouvoir
  déterminer si $N'=50\,000$ est dans 
  l'intervalle de confiance ou non. 

  On doit donc imposer à la borne supérieure de l'intervalle de
  confiance: 
  \[\begin{array}{ll}
  \dfrac{500}{50\,000}\leqslant 0,048+\dfrac{1}{\sqrt{n}}
  &\iff
  \dfrac{500}{50\,000}-0,048\leqslant \dfrac{1}{\sqrt{n}} \\[0.5cm]
  &\iff
  \dfrac{1}{\dfrac{500}{50\,000}-0,048}\leqslant \sqrt{n} \\[0.8cm]
  &\iff
  \lp\dfrac{1}{\dfrac{500}{50\,000}-0,048}\rp^2\leqslant n
  \end{array}\]
  soit, $n\geqslant 692$: il faudrait donc prélever un échantillon d'au
  moins 692 poissons. 
\end{enumerate}



\end{document}

Télécharger le fichier source Latex